Stay ahead of learning milestones! Enroll in a class over the summer!

G
Topic
First Poster
Last Poster
A geometry problem involving 2 circles
Ujiandsd   1
N 18 minutes ago by Ujiandsd
Source: L
Point M is the midpoint of side BC of triangle ABC. The length of the radius of the outer circle of triangle ABM, triangle ACM
is 5 and 7 respectively find the distance between the center of their outer circles
1 reply
Ujiandsd
May 11, 2025
Ujiandsd
18 minutes ago
Thailand MO 2025 P2
Kaimiaku   0
31 minutes ago
A school sent students to compete in an academic olympiad in $11$ differents subjects, each consist of $5$ students. Given that for any $2$ different subjects, there exists a student compete in both subjects. Prove that there exists a student who compete in at least $4$ different subjects.
0 replies
+1 w
Kaimiaku
31 minutes ago
0 replies
Thailand MO 2025 P3
Kaimiaku   2
N 34 minutes ago by lbh_qys
Let $a,b,c,x,y,z$ be positive real numbers such that $ay+bz+cx \le az+bx+cy$. Prove that $$ \frac{xy}{ax+bx+cy}+\frac{yz}{by+cy+az}+\frac{zx}{cz+az+bx} \le \frac{x+y+z}{a+b+c}$$
2 replies
Kaimiaku
an hour ago
lbh_qys
34 minutes ago
Burapha integer
EeEeRUT   1
N an hour ago by ItzsleepyXD
Source: TMO 2025 P1
For each positive integer $m$, denote by $d(m)$ the number of positive divisors of $m$. We say that a positive integer $n$ is Burapha integer if it satisfy the following condition
[list]
[*] $d(n)$ is an odd integer.
[*] $d(k) \leqslant d(\ell)$ holds for every positive divisor $k, \ell$ of $n$, such that $k < \ell$
[/list]
Find all Burapha integer.
1 reply
EeEeRUT
an hour ago
ItzsleepyXD
an hour ago
Algebra inequalities
TUAN2k8   1
N an hour ago by lbh_qys
Source: Own
Is that true?
Let $a_1,a_2,...,a_n$ be real numbers such that $0 \leq a_i \leq 1$ for all $1 \leq i \leq n$.
Prove that: $\sum_{1 \leq i<j \leq n} (a_i-a_j)^2 \leq \frac{n}{2}$.
1 reply
TUAN2k8
an hour ago
lbh_qys
an hour ago
Quadrilateral with Congruent Diagonals
v_Enhance   37
N an hour ago by Ilikeminecraft
Source: USA TSTST 2012, Problem 2
Let $ABCD$ be a quadrilateral with $AC = BD$. Diagonals $AC$ and $BD$ meet at $P$. Let $\omega_1$ and $O_1$ denote the circumcircle and the circumcenter of triangle $ABP$. Let $\omega_2$ and $O_2$ denote the circumcircle and circumcenter of triangle $CDP$. Segment $BC$ meets $\omega_1$ and $\omega_2$ again at $S$ and $T$ (other than $B$ and $C$), respectively. Let $M$ and $N$ be the midpoints of minor arcs $\widehat {SP}$ (not including $B$) and $\widehat {TP}$ (not including $C$). Prove that $MN \parallel O_1O_2$.
37 replies
v_Enhance
Jul 19, 2012
Ilikeminecraft
an hour ago
geometry
EeEeRUT   1
N an hour ago by ItzsleepyXD
Source: TMO 2025
Let $D,E$ and $F$ be touch points of the incenter of $\triangle ABC$ at $BC, CA$ and $AB$, respectively. Let $P,Q$ and $R$ be the circumcenter of triangles $AFE, BDF$ and $CED$, respectively. Show that $DP, EQ$ and $FR$ concurrent.
1 reply
EeEeRUT
an hour ago
ItzsleepyXD
an hour ago
Spanish Mathematical Olympiad 2002, Problem 1
OmicronGamma   3
N an hour ago by NicoN9
Source: Spanish Mathematical Olympiad 2002
Find all the polynomials $P(t)$ of one variable that fullfill the following for all real numbers $x$ and $y$:
$P(x^2-y^2) = P(x+y)P(x-y)$.
3 replies
OmicronGamma
Jun 2, 2017
NicoN9
an hour ago
Additive set with special property
the_universe6626   1
N 2 hours ago by jasperE3
Source: Janson MO 1 P2
Let $S$ be a nonempty set of positive integers such that:
$\bullet$ if $m,n\in S$ then $m+n\in S$.
$\bullet$ for any prime $p$, there exists $x\in S$ such that $p\nmid x$.
Prove that the set of all positive integers not in $S$ is finite.

(Proposed by cknori)
1 reply
the_universe6626
Feb 21, 2025
jasperE3
2 hours ago
ISI UGB 2025 P4
SomeonecoolLovesMaths   8
N 2 hours ago by chakrabortyahan
Source: ISI UGB 2025 P4
Let $S^1 = \{ z \in \mathbb{C} \mid |z| =1 \}$ be the unit circle in the complex plane. Let $f \colon S^1 \longrightarrow S^2$ be the map given by $f(z) = z^2$. We define $f^{(1)} \colon = f$ and $f^{(k+1)} \colon = f \circ f^{(k)}$ for $k \geq 1$. The smallest positive integer $n$ such that $f^{(n)}(z) = z$ is called the period of $z$. Determine the total number of points in $S^1$ of period $2025$.
(Hint : $2025 = 3^4 \times 5^2$)
8 replies
SomeonecoolLovesMaths
Sunday at 11:24 AM
chakrabortyahan
2 hours ago
So Many Terms
oVlad   7
N 3 hours ago by NuMBeRaToRiC
Source: KöMaL A. 765
Find all functions $f:\mathbb{R}\to\mathbb{R}$ which satisfy the following equality for all $x,y\in\mathbb{R}$ \[f(x)f(y)-f(x-1)-f(y+1)=f(xy)+2x-2y-4.\]Proposed by Dániel Dobák, Budapest
7 replies
oVlad
Mar 20, 2022
NuMBeRaToRiC
3 hours ago
IMO ShortList 2001, geometry problem 2
orl   49
N Apr 11, 2025 by Nari_Tom
Source: IMO ShortList 2001, geometry problem 2
Consider an acute-angled triangle $ABC$. Let $P$ be the foot of the altitude of triangle $ABC$ issuing from the vertex $A$, and let $O$ be the circumcenter of triangle $ABC$. Assume that $\angle C \geq \angle B+30^{\circ}$. Prove that $\angle A+\angle COP < 90^{\circ}$.
49 replies
orl
Sep 30, 2004
Nari_Tom
Apr 11, 2025
IMO ShortList 2001, geometry problem 2
G H J
Source: IMO ShortList 2001, geometry problem 2
The post below has been deleted. Click to close.
This post has been deleted. Click here to see post.
orl
3647 posts
#1 • 7 Y
Y by mathmaths, r0518, Math-Ninja, Adventure10, HWenslawski, ImSh95, Mango247
Consider an acute-angled triangle $ABC$. Let $P$ be the foot of the altitude of triangle $ABC$ issuing from the vertex $A$, and let $O$ be the circumcenter of triangle $ABC$. Assume that $\angle C \geq \angle B+30^{\circ}$. Prove that $\angle A+\angle COP < 90^{\circ}$.
Attachments:
This post has been edited 2 times. Last edited by orl, Oct 25, 2004, 12:05 AM
Z K Y
The post below has been deleted. Click to close.
This post has been deleted. Click here to see post.
orl
3647 posts
#2 • 6 Y
Y by r0518, Math-Ninja, alireza_32, Adventure10, ImSh95, Mango247
Please post your solutions. This is just a solution template to write up your solutions in a nice way and formatted in LaTeX. But maybe your solution is so well written that this is not required finally. For more information and instructions regarding the ISL/ILL problems please look here: introduction for the IMO ShortList/LongList project and regardingsolutions :)
Z K Y
The post below has been deleted. Click to close.
This post has been deleted. Click here to see post.
mathmanman
1444 posts
#3 • 6 Y
Y by Math-Ninja, Not_real_name, myh2910, Adventure10, ImSh95, Mango247
Like usual, we let : $\angle A = \alpha$, $\angle B = \beta$ and $\angle C = \gamma$.
Then, $\angle BCO = \frac 12(\pi - \angle BOC) = \frac{\pi}2 - \frac 12(2 \alpha)$.
Since $\triangle ABC$ is acute-angled, $O$ is inside the triangle, and $P$ is between $B$ and $C$. So, we just have to prove that $\angle COP = \angle POC$, or, equivalently : $PO > PC$. Let $OC = R$, then $b = 2R \sin{\beta}$ (thanks to the sine law).
Since $PC = b \cos{\gamma}$, we have :
\begin{eqnarray*}PC^2 < PO^2 &\iff& PC^2 < R^2 + PC^2 - 2Rb \cos{\gamma} \cos{(\frac{\pi}2 - \alpha)} \\ &\iff& 0 < \frac{b^2}{4 \sin^2{\beta}} - \frac{b^2 \cos{\gamma} \sin{\alpha}}{\sin{\beta}} \\ &\iff& 4 \sin{\alpha} \sin{\beta} \cos{\gamma} < 1 \\ &\iff& 2 \sin{\alpha}(\sin{(\beta + \gamma)} - \sin{(\gamma - \beta)}) <1 \\ &\Longleftarrow& 2 \sin{\alpha}(\sin{\alpha} - \sin{\frac{\pi}6}) \le 1 \\ &\Longleftarrow& 2 \sin{\alpha}\left(1 - \frac 12 \right) \le 1 \\ &\Longleftarrow& 2 - 2 \left(\frac 12 \right) \le 1 \end{eqnarray*}
which is true.
Z K Y
The post below has been deleted. Click to close.
This post has been deleted. Click here to see post.
yetti
2643 posts
#4 • 1 Y
Y by Adventure10
The problem condition $C\ge B+30^\circ$ is equivalent to

$\sin(C-B) \ge \sin 30^\circ = \frac{1}{2}\ \Longleftrightarrow\ \sin C\cos B-\sin B\cos C\ge \frac{1}{2}\ \Longleftrightarrow$

$\frac{c}{2R}\cdot \frac{c^{2}+a^{2}-b^{2}}{2ca}-\frac{b}{2R}\cdot \frac{a^{2}+b^{2}-c^{2}}{2ab}\ge \frac{1}{2}\ \Longleftrightarrow\ c^{2}-b^{2}\ge aR$

The angles $A+\angle COP$ add to $90^\circ$ in an acute $\triangle ABC,$ iff the triangle $\triangle COP$ is isosceles with $\angle COP = \angle OCP,$ because it is always trus that $\angle OCP \equiv \angle OCB = 90^\circ-A$. Thus $A+\angle COP < 90^\circ$ is equivalent to $PO > PC.$ Let M be the midpoint of BC. By Pythagorean theorem for the right triangle $\triangle OMP,$

$PO^{2}= OM^{2}+MP^{2}$

$MP = CM-CP,\ \ CM = \frac{a}{2},\ \ CP = b \cos C = \frac{a^{2}+b^{2}-c^{2}}{2a}$

$PO > PC\ \Longleftrightarrow\ OM^{2}+(CM-CP)^{2}> CP^{2}\ \Longleftrightarrow$

$OM^{2}+CM^{2}-2CM \cdot CP > 0\ \Longleftrightarrow OM^{2}+\frac{a^{2}}{4}-\frac{a^{2}+b^{2}-c^{2}}{2}> 0\ \Longleftrightarrow$

$OM^{2}+\frac{c^{2}-b^{2}}{2}> \frac{a^{2}}{4}$

The consequence $c^{2}-b^{2}\ge aR$ of the prolem condition and the trivial facts $OM > 0,$ $R > \frac{a}{2}$ for an acute triangle imply

$OM^{2}+\frac{c^{2}-b^{2}}{2}\ge OM^{2}+\frac{aR}{2}> OM^{2}+\frac{a^{2}}{4}> \frac{a^{2}}{4}$

hence $A+\angle COP < 90^\circ$.
Z K Y
The post below has been deleted. Click to close.
This post has been deleted. Click here to see post.
BenasKikutis
3 posts
#5 • 3 Y
Y by Lukaluce, Adventure10, Mango247
Can anybody post solution without trigonometry, if there is one. Please
Z K Y
The post below has been deleted. Click to close.
This post has been deleted. Click here to see post.
yetti
2643 posts
#6 • 2 Y
Y by Adventure10, Mango247
$ AP, AO$ and parallel to $ BC$ through $ A$ cut $ (O)$ again at $ X, Y, Z,$ respectively. $ YZ$ cuts $ BC$ at $ Q.$ $ XY \parallel BC,$ $ YX \perp AX$ $ \Longrightarrow$ $ AXYZ$ is rectangle, $ XY = PQ = AZ$ and $ M$ is common midpoint of $ BC, PQ.$ For any acute $ \triangle ABC,$ $ OM > 0,$ $ R > \frac {BC}{2}$ and

$ \angle COP < \angle OCP = 90^\circ - \angle A$ $ \Longleftrightarrow$ $ CP^2 < OP^2$ $ \Longleftrightarrow$ $ \frac {(BC - PQ)^2}{4} < OM^2 + \frac {PQ^2}{4}$ $ \Longleftrightarrow$ $ OM^2 + \frac {BC \cdot PQ}{2} > \frac {BC^2}{4}$

From $ \angle XAY = \angle C - \angle B \ge 30^\circ$ $ \Longrightarrow$ $ PQ = XY \ge \frac {AY}{2} = R.$ Therefore,

$ OM^2 + \frac {BC \cdot PQ}{2} \ge OM^2 + \frac {BC \cdot R}{2} > \frac {BC^2}{4}$ $ \Longrightarrow$ $ CP < OP$ $ \Longrightarrow$ $ \angle COP + \angle A < 90^\circ.$
Z K Y
The post below has been deleted. Click to close.
This post has been deleted. Click here to see post.
math154
4302 posts
#7 • 2 Y
Y by Adventure10, Mango247
Blah here's my silly way to show $OP>CP$.

From the power of point $P$ we see that $R^2-OP^2=BP\cdot CP$, so it is equivalent to show that
\[OP>\frac{R^2-OP^2}{BP}=CP,\]or by the Law of Cosines on $\angle{BPO}$, that
\begin{align*}
OP\cdot BP+OP^2 &> R^2 = BO^2 = BP^2+OP^2-2BP\cdot OP\cos\angle{BPO}\\
\Longleftrightarrow OP+OP(2\cos\angle{BPO}) &> BP.
\end{align*}But by the Law of Sines on $\triangle{APO}$, we have
\[2\cos\angle{BPO} = 2\sin\angle{APO} = 2\frac{AO\sin(C-B)}{OP} \ge \frac{AO}{OP}.\]Hence
\[OP+OP(2\cos\angle{BPO}) \ge OP+AO = OP+BO > BP\]by the Triangle Inequality, so we're done.
Z K Y
The post below has been deleted. Click to close.
This post has been deleted. Click here to see post.
golem24
92 posts
#8 • 3 Y
Y by samrocksnature, Nafis_Noor, Adventure10
Draw $OK\perp AP$ and $OD\perp BC$.Obviously $KOPD$ is a rectangle.
$AO=CO$ means $\triangle ABC$ is isosceles. So $\angle CAO=\frac{1}{2}(\pi-\angle AOC)=\frac{\pi}{2}-\angle B$

[asy]
/* File unicodetex not found. */

 /* Geogebra to Asymptote conversion, documentation at artofproblemsolving.com/Wiki, go to User:Azjps/geogebra */
import graph; size(11.74cm); 
real labelscalefactor = 0.5; /* changes label-to-point distance */
pen dps = linewidth(0.7) + fontsize(10); defaultpen(dps); /* default pen style */ 
pen dotstyle = black; /* point style */ 
real xmin = -4.3, xmax = 7.44, ymin = -2.64, ymax = 6.3;  /* image dimensions */

 /* draw figures */
draw(circle((1.82,1.42), 3.76)); 
draw((0.29,4.86)--(-1.59,-0.16)); 
draw((-1.59,-0.16)--(5.21,-0.21)); 
draw((5.21,-0.21)--(0.29,4.86)); 
draw((0.29,4.86)--(0.27,-0.17)); 
draw((1.82,1.42)--(0.28,1.44)); 
draw((1.82,1.42)--(-1.59,-0.16)); 
draw((1.82,1.42)--(0.27,-0.17)); 
draw((1.82,1.42)--(1.8,-0.19)); 
 /* dots and labels */
dot((1.82,1.42),dotstyle); 
label("$O$", (2,1.46), NE * labelscalefactor); 
dot((0.29,4.86),dotstyle); 
label("$A$", (0.38,4.98), NE * labelscalefactor); 
dot((-1.59,-0.16),dotstyle); 
label("$C$", (-1.42,-0.38), NE * labelscalefactor); 
dot((5.21,-0.21),dotstyle); 
label("$B$", (4.78,-0.52), NE * labelscalefactor); 
dot((0.27,-0.17),dotstyle); 
label("$P$", (0.24,-0.52), NE * labelscalefactor); 
dot((0.28,1.44),dotstyle); 
label("$H$", (0.36,1.56), NE * labelscalefactor); 
dot((1.8,-0.19),dotstyle); 
label("$D$", (1.74,-0.48), NE * labelscalefactor); 
clip((xmin,ymin)--(xmin,ymax)--(xmax,ymax)--(xmax,ymin)--cycle); 
 /* end of picture */
[/asy]

$\angle CAP=\frac{\pi}{2}-\angle C$, so angle $PAO=\angle CAO-\angle CAP=\angle C-\angle B\ge 30^\circ$.


So we have $KO\ge \frac{1}{2}AO$(You may verify this using sine rule as well).As $KOPD$ is a rectangle $KO=PD>\frac{1}{2}AO=\frac{1}{2}CO>\frac{1}{2}CD$. So $PC<PD<OP\implies \angle COP<\angle PCO=\angle OCD \dots (*)$.

We now have $\angle COP+ \angle A=\angle COP+\angle COD<\angle OCD+\angle COD=90^\circ$.
Z K Y
The post below has been deleted. Click to close.
This post has been deleted. Click here to see post.
hyperspace.rulz
287 posts
#9 • 2 Y
Y by Adventure10, Mango247
Another approach that utilises power of a point:

As in all the other solutions here, we establish that we need to prove $OP>CP$. This in turn is equivalent, obviously, to:

$OP^2>CP^2$
$\Leftrightarrow R^2-PC \cdot PB>CP^2$

This is because of Power of a Point. We have that this is in turn equivalent to:

$R^2>CP^2+PC \cdot PB=CP(CP+PB)=CP \cdot CB$

Now note that $PB=AB\cos B=2R\sin C\cos B$, and similarly $PC=2R\sin B\cos C$. It follows that $PB-PC=2R\sin(C-B) \geq R$, where the last step follows from the problem's condition. Since $PB+PC=BC$, we deduce that $PB \geq \frac{R+BC}{2}$, and hence $PC \leq \frac{BC-R}{2}$. So we have:

$CP \cdot CB \leq \frac{BC(BC-R)}{2}$, so it suffices to show that:

$\frac{BC(BC-R)}{2}<R^2$
$\Leftrightarrow \frac{a(a-R)}{2}<R^2$
where $BC=a$. Bearing in mind that $a<2R$, we have:

$a-R<R$
$\Leftrightarrow (a-R)(a+R)<R(a+R)$
$\Leftrightarrow a^2-R^2<aR+R^2$
$\Leftrightarrow a^2-aR<2R^2$
$\Leftrightarrow a(a-R)<2R^2$
$\Leftrightarrow \frac{a(a-R)}{2}<R^2$

as desired.
Z K Y
The post below has been deleted. Click to close.
This post has been deleted. Click here to see post.
JSGandora
4216 posts
#10 • 2 Y
Y by Taco12, Adventure10
Extend $CO$ to meet the circumcircle of $ABC$ at $D$. Then $\angle DAB+\angle A=90^\circ$ and we want to prove that
\[\angle A+\angle COP<90^\circ=\angle DAB+\angle A\Longleftrightarrow \angle COP<\angle DAB.\]
We also have $\angle DAB=\angle BCD$ so what we want to prove is equivalent to proving
\[\angle COP<\angle BCD=\angle PCO\]
which is also equivalent to proving $OP>CP$.

We have
\begin{align*}
OP&>CP\\
\Longleftrightarrow OP^2&>CP^2 \\
\Longleftrightarrow CP^2+OC^2-2(CO)(CP)\cos\angle PCO&>CP^2\\
\Longleftrightarrow OC^2&>2(OC)(CP)\cos\angle PCO \\
\Longleftrightarrow OC&>2(CP)\cos \angle PCO
\end{align*}
Now let $\angle B=\beta $ and $\angle C=\gamma$, then $\angle PCO=\beta+\gamma-90$. Also, $PC=b\cos \gamma$ and $OC=\frac{b}{2\sin \beta}$. Now what we want to prove is
\[\frac {b}{2\sin \beta}>2b\cos \gamma\cos(\beta+\gamma-90)=2b\cos\gamma\sin(\beta+\gamma)\]
or
\[1>4\sin\beta\cos\gamma\sin(\beta+\gamma).\]
By the condition, $\gamma\geq \beta+30$ and the triangle is acute so $\cos\gamma<\cos(\beta+30)$. Then
\[4\sin\beta\cos\gamma\sin(\beta+\gamma)<4\sin\beta\cos(\beta+30)=2\sin\beta(\cos\beta\sqrt3-\sin\beta).\]
Taking the derivative of the RHS we get
\[2(\cos \beta(\cos\beta\sqrt3-\sin\beta)+\sin\beta(-\sin\beta\sqrt3-\cos\beta)) \\ =2(\sqrt{3}(\cos^2\beta-\sin^2\beta-2\sin\beta\cos\beta))=2(\sqrt3\cos2\beta-\sin2\beta)\]
We find the critical points:
\begin{align*}
2(\sqrt3\cos2\beta-\sin2\beta)&=0 \\
\sqrt{3}\cos 2\beta=\sin2\beta
\end{align*}
Since $\beta\neq 45^\circ$, the LHS is not equal to 0 so we divide
\[\tan 2\beta=\sqrt{3}\]
and the only solution to this is such that $0^\circ<\beta<60^\circ$ $2\beta=60\circ\implies \beta=30^\circ$. Thus the critical points are $\beta=\{0^\circ,30^\circ,60^circ\}$. Plugging the two critical points in, we have
\[2\sin\beta(\cos\beta\sqrt3-\sin\beta)=0\]
for $\beta=0^\circ$ and $\beta=60^\circ$ and
\[2\sin\beta(\cos\beta\sqrt3-\sin\beta)=1\]
when $\beta=30^\circ$ therefore the maximum is achieved when $\beta=30^\circ$ however that means $\gamma =90^\circ$ which is impossible since the triangle is acute so therefore the maximum is not achievable and thus
\[4\sin\beta\cos\gamma\sin(\beta+\gamma)<1\]
as desired. $\blacksquare$
Z K Y
The post below has been deleted. Click to close.
This post has been deleted. Click here to see post.
sayantanchakraborty
505 posts
#11 • 1 Y
Y by Adventure10
Outlines of my proof

Let $\angle{COP}=\theta$.Easy to show that its acute.
Sine rule in $\triangle{COP}$ gives
$\frac{R}{CP}=\frac{cos(A-\theta)}{sin\theta} \Rightarrow cosA cot\theta+sinA=\frac{1}{2sinBcosC}=\frac{1}{sinA-sin(C-B)} \ge \frac{1}{sinA-\frac{1}{2}}=\frac{2}{2sinA-1}$.

From here we get the upper bound of $\tan\theta$.Now it is easy(using some elementary calcuations) to show that $tanA tan\theta <1$ so that $tan{A+\theta}>0$ and the result follows...
Z K Y
The post below has been deleted. Click to close.
This post has been deleted. Click here to see post.
CanVQ
83 posts
#12 • 2 Y
Y by Adventure10, Mango247
orl wrote:
Consider an acute-angled triangle $ABC$. Let $P$ be the foot of the altitude of triangle $ABC$ issuing from the vertex $A$, and let $O$ be the circumcenter of triangle $ABC$. Assume that $\angle C \geq \angle B+30^{\circ}$. Prove that $\angle A+\angle COP < 90^{\circ}$.
My solution:
Click to reveal hidden text
Let $D$ be the foot of the altitude of $\triangle ABC$ issuing from $B.$ It is known that $DP \perp OC$ and $\triangle CPD \sim \triangle CAP.$

Let $E=OC \cap DP,$ we have \[\cos A=\cos \angle CPE=\frac{PE}{PC}\] and \[\sin \angle COP=\frac{PE}{OP}.\] Therefore, \[\angle A+\angle COP<90^{\circ} \Leftrightarrow \sin \angle COP<\sin (90^{\circ}-\angle A)=\cos A\Leftrightarrow OP>CP.\] Now, let us compute the lengths of $OP$ and $CP.$ Let $M$ be the midpoint of $BD$ and $R$ be the radius of $(O),$ we have \[CP=AC\cdot \cos \angle ACP=2R\cdot \sin B\cos C\] and \[CM=OC\cdot \sin COM=R\cdot \sin A,\quad OM=OC\cdot \cos \angle COM=R\cdot \cos A.\] Therefore, \[\begin{aligned} OP^2&=OM^2+PM^2=OM^2+(CM-CP)^2\\ &=R^2\big[ \cos^2A+(\sin A- 2\sin B\cos C)^2\big].\end{aligned}\] It suffices to prove that \[4\sin^2B\cos^2C<\cos^2A+(\sin A-2\sin B\cos C)^2,\] or \[1>4\sin A\sin B\cos C.\] We have \[\begin{aligned} 4\sin A\sin B\cos C&=2\sin A \big[ \sin A-\sin (C-B)\big] \le 2\sin A(\sin A-\sin 30^{\circ})\\ &=2\sin^2A-\sin A =1+(\sin A-1)(2\sin A+1)<1.\end{aligned}\] The proof is done.
Attachments:
Z K Y
The post below has been deleted. Click to close.
This post has been deleted. Click here to see post.
guldam
31 posts
#13 • 3 Y
Y by Adventure10, Mango247, and 1 other user
I hope this solution would not be duplicate with any other presented above.

Since $\angle PCO = 90 - \angle A$ it is well noted that to prove $PO > PC $ is enough.

It is also well-known that $\angle OAP = \angle C - \angle B \ge 30$

Let point $M$ to be the midpoint of $BC$ then

$MP = R\sin{\angle OAP} \>= R \sin{30} = \frac{R}{2}$

since $ 30\le \angle OAP < \angle A \le 90$

$ R < a = 2R \sin{A} < 2R $

therefore,

$ PC = MC - MP \le \frac{a-R}{2} < \frac{R}{2} \le MP < PO $ $Q.E.D$

the last inequality hold since $PO$ is the hypotenuse of the right triangle $\triangle PMO$
Z K Y
The post below has been deleted. Click to close.
This post has been deleted. Click here to see post.
TelvCohl
2312 posts
#14 • 3 Y
Y by jonathan3951, guptaamitu1, Adventure10
My solution:

Let $ A' $ be the reflection of $ A $ in the perpendicular bisector of $ BC $ .
Let $ M, N $ be the midpoint of $ BC, AA' $, respectively .

Since $ \angle AOA'=2(\angle ACB-\angle CBA) \geq 60^{\circ} $ ,
so we get $ AA' \geq OA \Longrightarrow MP=AN \geq \tfrac{1}{2} OA $ ,
hence $ CP=MC-MP<OC-MP \leq \tfrac{1}{2} OA \leq MP \Longrightarrow \angle COP<\angle OCB $
$ \Longrightarrow \angle BAC+\angle COP=\angle MOC+\angle COP<\angle MOC+\angle OCB=90^{\circ} $

Q.E.D
Z K Y
The post below has been deleted. Click to close.
This post has been deleted. Click here to see post.
va2010
1276 posts
#15 • 3 Y
Y by Adventure10, Mango247, shafikbara48593762
Here's a solution, from working with swimmer:

We just have to show $PO > PC$. But, using the following diagram:

https://hostr.co/file/J105WwHiuEme/IMOGEO.png

we can see that $PO > FO > EF > CP$, so we are done.
Z K Y
G
H
=
a